2012 AMC 12A Problems/Problem 24

Revision as of 19:28, 18 February 2012 by Aplus95 (talk | contribs) (Created page with "== Problem == Let <math>\{a_k\}_{k=1}^{2011}</math> be the sequence of real numbers defined by <math>a_1=0.201,</math> <math>a_2=(0.2011)^{a_1},</math> <math>a_3=(0.20101)^{a_2}...")
(diff) ← Older revision | Latest revision (diff) | Newer revision → (diff)

Problem

Let $\{a_k\}_{k=1}^{2011}$ be the sequence of real numbers defined by $a_1=0.201,$ $a_2=(0.2011)^{a_1},$ $a_3=(0.20101)^{a_2},$ $a_4=(0.201011)^{a_3}$, and in general,

\[a_k=\begin{cases}(0.\underbrace{20101\cdots 0101}_{k+2\text{ digits}})^{a_{k-1}}\qquad\text{if }k\text{ is odd,}\\(0.\underbrace{20101\cdots 01011}_{k+2\text{ digits}})^{a_{k-1}}\qquad\text{if }k\text{ is even.}\end{cases}\]

Rearranging the numbers in the sequence $\{a_k\}_{k=1}^{2011}$ in decreasing order produces a new sequence $\{b_k\}_{k=1}^{2011}$. What is the sum of all integers $k$, $1\le k \le 2011$, such that $a_k=b_k?$

$\textbf{(A)}\ 671\qquad\textbf{(B)}\ 1006\qquad\textbf{(C)}\ 1341\qquad\textbf{(D)}\ 2011\qquad\textbf{(E)}\2012$ (Error compiling LaTeX. Unknown error_msg)


Solution